Course Content
All Previous Years Krok 2 Papers with Explanations
About Lesson


Question From ( 51 To 100 )


51. A 38 y.o. patient has been treated in a hospital. A fever of 390C, chest pain which is worsened by breathing, cough, brownish sputum appeared on the 7-th day of the treatment. Chest X- ray shows left lower lobe infiltrate. Which of the following is the treatment of choice for this patient?

A. Cephalosporins of the III generation

B. Penicillin

C. Erythromycin

D. Tetracycline

E. Streptomycin


Answer: Cephalosporins of the III generation

Explanation

The patient’s presentation of fever, chest pain worsened by breathing, cough, and brownish sputum, along with the chest X-ray findings of a left lower lobe infiltrate suggest a diagnosis of community-acquired pneumonia.   Community-acquired pneumonia is typically caused by bacteria, such as Streptococcus pneumoniae, Haemophilus influenzae, and atypical pathogens, such as Mycoplasma pneumoniae and Chlamydophila pneumoniae.

The choice of antibiotic therapy for community-acquired pneumonia depends on the severity of the illness, age of the patient, and risk factors for resistant pathogens.   In this case, the patient is experiencing symptoms of severe pneumonia, which suggests the need for empirical treatment with broad-spectrum antibiotics, such as cephalosporins of the III generation. These antibiotics have good activity against the most common pathogens causing community-acquired pneumonia, including Streptococcus pneumoniae and Haemophilus influenzae.  

Penicillin (option B) is not the treatment of choice for community-acquired pneumonia as it has limited activity against atypical pathogens and may be less effective against penicillin-resistant strains of Streptococcus pneumoniae.   Erythromycin (option C) is an alternative antibiotic for the treatment of community-acquired pneumonia, particularly in patients who are allergic to penicillin. However, cephalosporins of the III generation are preferred in severe cases.  

Tetracycline (option D) is not the first-line treatment for community-acquired pneumonia due to its limited activity against Streptococcus pneumoniae.   Streptomycin (option E) is an aminoglycoside antibiotic that is used to treat infections caused by gram-negative bacteria, but it is not the treatment of choice for community-acquired pneumonia.   Therefore, based on the patient’s presentation and the chest X-ray findings, the most appropriate choice of antibiotic therapy for this patient is cephalosporins of the III generation.


52. A 62 y.o. patient suffers from DM-2. Diabetes is being compensated by diet and Maninilum. Patient has to undergo an operation on inguinal hernia. What tactics of hypoglycemic therapy should be used?

A. Prescribe fast-acting insulin

B. Give Glurenorm in place of Maninilum

C. Continue with the current therapy

D. Prescribe long-acting insulin

E. Prescribe guanyl guanidines


Answer: Prescribe fast-acting insulin

Explanation

The 62-year-old patient with type 2 diabetes who is being treated with diet and Maninilum (an oral sulfonylurea) is scheduled to undergo an operation on inguinal hernia. During and after the surgery, there is a risk of hypoglycemia due to the stress response and altered metabolism associated with surgery.

Therefore, it is important to adjust the patient’s hypoglycemic therapy to prevent hypoglycemia during and after the surgery.   Continuing with the current therapy (option C) may not be sufficient to prevent hypoglycemia during and after the surgery, as the stress response and altered metabolism associated with surgery can affect glycemic control.  

Giving Glurenorm in place of Maninilum (option B) may not be effective in preventing hypoglycemia during and after the surgery, as Glurenorm is also an oral hypoglycemic agent that can cause hypoglycemia.   Prescribing long-acting insulin (option D) may not be appropriate in this case, as it may not provide adequate glycemic control during and after the surgery, and it may increase the risk of hypoglycemia.  

Prescribing guanyl guanidines (option E) is not appropriate in this case, as guanyl guanidines (e.g., metformin) are not fast-acting agents and may not provide immediate glycemic control during and after the surgery.  

Therefore, the most appropriate tactic of hypoglycemic therapy for this patient is to prescribe fast-acting insulin, which can be administered as a bolus or infusion to maintain glycemic control during and after the surgery. The dose of insulin should be adjusted based on the patient’s blood glucose levels and other clinical factors. Close monitoring of blood glucose levels and clinical status is essential to prevent hypoglycemia and ensure optimal glycemic control during and after the surgery.


53. A patient of 32 y.o. complains of severe weakness, tremor of extremities. Objective examination: body weight loss, wet and warm skin. The thyroid gland is enlarged up to the 3-rd degree, painless, elastic. Ps108/min. BP- 160/55 mm Hg. There are no other abnormalties. The diagnosis is:

A. Diffuse toxic goiter of the 3-rd degree, thyrotoxicosis of the average degree

B. Diffuse euthyroid goiter of the 3-rd degree

C. Chronic autoimmune thyroiditis, hypertrophic type

D. Chronic fibrous thyroiditis

E. Toxiferous adenoma of the thyroid gland


Answer: Diffuse toxic goiter of the 3-rd degree, thyrotoxicosis of the average degree

Explanation

The patient’s presentation of severe weakness, tremor of extremities, weight loss, wet and warm skin, palpable and enlarged thyroid gland, and tachycardia (heart rate of 108 beats per minute) suggest the diagnosis of diffuse toxic goiter (also known as Graves’ disease) with thyrotoxicosis.  

Graves’ disease is an autoimmune disorder that causes overproduction of thyroid hormones, resulting in hyperthyroidism. The enlarged thyroid gland, or goiter, is due to the stimulation of the thyroid gland by thyroid-stimulating antibodies produced in the autoimmune process. The severity of the goiter is classified into three degrees based on its size and visibility.  

The patient’s symptoms and clinical findings suggest a diffuse toxic goiter of the 3rd degree, which is characterized by a visibly enlarged thyroid gland that extends beyond the borders of the neck, along with signs of thyrotoxicosis.   Diffuse euthyroid goiter (option B) is an enlarged thyroid gland without any signs of thyroid hormone excess or deficiency.

The patient in this case is presenting with symptoms and clinical findings consistent with thyrotoxicosis.   Chronic autoimmune thyroiditis, hypertrophic type (option C) and chronic fibrous thyroiditis (option D) are both types of chronic inflammation of the thyroid gland, but they do not cause hyperthyroidism or an enlarged thyroid gland with signs of thyrotoxicosis.  

Toxic adenoma of the thyroid gland (option E) is a type of hyperthyroidism caused by a benign thyroid nodule that produces excessive amounts of thyroid hormone. However, toxic adenomas do not typically cause diffuse enlargement of the thyroid gland.  

Therefore, based on the patient’s symptoms and clinical findings, the most likely diagnosis is diffuse toxic goiter of the 3rd degree with thyrotoxicosis of the average degree. Appropriate management should be initiated promptly to control the hyperthyroidism and prevent complications.


54. A triad of symptoms (“stearing spot”, “terminal film”, “blood dew”) have been revealed on examination of a patient. What disease should you think about?

A. Psoriasis

B. Lichen ruber planus

C. Vasculitis

D. Seborrhea

E. Ritter’s disease


Answer:  Psoriasis

Explanation

The triad of symptoms described (“stearing spot”, “terminal film”, “blood dew”) is associated with psoriasis, a chronic autoimmune skin disorder characterized by the development of scaly, red, and itchy patches of skin.   The “steering spot” refers to the initial lesion of psoriasis, which is a small, red, scaly papule or plaque. The “terminal film” refers to the silvery-white scale that forms on the surface of the lesion. The “blood dew” refers to the bleeding that can occur when the scale is removed.  

Lichen ruber planus (option B) is a skin disorder characterized by itchy, flat-topped, purple bumps on the skin. It does not typically present with the triad of symptoms described.   Vasculitis (option C) is a group of disorders characterized by inflammation of the blood vessels, which can lead to damage to the organs and tissues supplied by the affected vessels.

Vasculitis does not present with the triad of symptoms described.   Seborrhea (option D) is a common skin disorder characterized by the overproduction of oil by the sebaceous glands, leading to the development of oily, red, and scaly patches of skin. It does not typically present with the triad of symptoms described.  

Ritter’s disease (option E), also known as staphylococcal scalded skin syndrome, is a rare and potentially life-threatening skin disorder caused by an exotoxin produced by Staphylococcus aureus. It is characterized by the development of large, fluid-filled blisters and peeling of the skin.

It does not typically present with the triad of symptoms described.   Therefore, based on the triad of symptoms described (“steering spot”, “terminal film”, “blood dew”), the most likely diagnosis is psoriasis. Appropriate management should be initiated to control the symptoms and prevent complications.


55. Patient 27 y.o. was hospitalized to the psychiatric hospital for the 4-th time during 2 years. Heard voices commenting on his actions, had delusions of persecution (was sure that the Mafia wanted to kill him). After a course of treatment with neuroleptics was discharged from hospital with the diagnosis of schizophrenia, state of remission. The secondary prevention of the relapses of schizophrenia requires:

A. Supportive treatment with neuroleptics of prolonged action

B. Long-term hospitalization

C. Psychiatric observation

D. Participation in a self-help group

E. Psychoanalytic treatment


Answer: Supportive treatment with neuroleptics of prolonged action

Explanation

Schizophrenia is a chronic mental disorder characterized by a combination of positive symptoms (such as hallucinations and delusions) and negative symptoms (such as social withdrawal and flattened affect). The disorder often requires long-term treatment and management to prevent relapses.  

In this case, the patient has been hospitalized multiple times for schizophrenia and was discharged with a diagnosis of schizophrenia in remission after treatment with neuroleptics. Therefore, the secondary prevention of relapses of schizophrenia requires ongoing treatment and management.  

Supportive treatment with neuroleptics of prolonged action (option A) is the most appropriate option for preventing relapses of schizophrenia. Neuroleptics, also known as antipsychotic medications, are the mainstay of treatment for schizophrenia and are effective in reducing the positive symptoms of the disorder. Prolonged-action neuroleptics can be administered on a long-term basis to prevent relapses and maintain remission.  

Long-term hospitalization (option B) is not necessary for all patients with schizophrenia and is usually reserved for those who require intensive treatment and management due to severe symptoms or risk of harm to self or others.   Psychiatric observation (option C) may be appropriate for some patients with schizophrenia, but it is not sufficient for preventing relapses of the disorder.  

Participation in a self-help group (option D) may be helpful for some patients with schizophrenia to provide social support and coping strategies, but it is not sufficient for preventing relapses of the disorder.   Psychoanalytic treatment (option E) is not typically used as the primary treatment for schizophrenia, as it focuses on exploring unconscious conflicts and emotions rather than addressing the specific symptoms of the disorder.  

Therefore, based on the patient’s history of schizophrenia and the need for secondary prevention of relapses, the most appropriate option is to provide supportive treatment with neuroleptics of prolonged action. Close monitoring and follow-up are also important to ensure optimal treatment and management of the disorder.


56. A 42 y.o. woman works at the factory on the fabrication of mercury thermometers, complains of the headache, swoons, reduction of memory, small and frequent flutter of fingers of drawn hands, the eyelids and the tongue, bleeding gums, gingivitis. What preparation is it nessesary to use for the elimination of mercury from the organism?

A. Unithiol

B. Pentoxil

C. Magnesium sulphate

D. Sodium hydrate of carbon

E. Seduxen


Answer:  Unithiol

Explanation

The patient’s symptoms are consistent with mercury poisoning, which can occur in individuals who are exposed to mercury in their occupation, such as in the fabrication of mercury thermometers. The symptoms of mercury poisoning can include headache, memory loss, tremors, gingivitis, and other neurological and gastrointestinal symptoms.   Unithiol (also known as dimercaptopropanol) is a chelating agent that is used to treat heavy metal poisoning, including mercury poisoning.

It works by binding to the mercury in the body and forming a complex that can be excreted in the urine. Unithiol is administered intravenously and is effective in removing mercury from the body.   Pentoxil (option B) is a medication that is used to improve blood flow and reduce inflammation in conditions such as peripheral vascular disease and diabetic neuropathy. It is not indicated for the treatment of heavy metal poisoning.  

Magnesium sulfate (option C) is a medication that is used to treat a variety of conditions, including eclampsia, pre-eclampsia, and magnesium deficiency. It is not indicated for the treatment of heavy metal poisoning.   Sodium bicarbonate (option D) is a medication that is used to treat acidosis and other gastrointestinal conditions. It is not indicated for the treatment of heavy metal poisoning.   Seduxen (option E) is a benzodiazepine medication that is used to treat anxiety, insomnia, and muscle spasms.

It is not indicated for the treatment of heavy metal poisoning.   Therefore, based on the patient’s symptoms and history of exposure to mercury, the most appropriate option for the elimination of mercury from the body is to administer Unithiol as a chelating agent. Close monitoring and follow-up are also important to ensure optimal treatment and management of the mercury poisoning.


57. The disease began acutely. The frequent watery stool developed 6 hours ago. The body’s temperature is normal. Then the vomiting was joined. On examination: his voice is hoarse, eyes are deeply sunken in the orbits. The pulse is frequent. Blood pressure is low. There is no urine. What is the preliminary diagnosis?

A. Cholera

B. Toxic food-borne infection

C. Salmonellosis

D. Dysentery

E. Typhoid fever


Answer: Cholera

Explanation

The symptoms described in the question, including acute onset of frequent watery diarrhea, vomiting, hoarse voice, sunken eyes, tachycardia (fast pulse), hypotension (low blood pressure), and decreased urine output, are consistent with cholera.  

Cholera is an acute diarrheal illness caused by the bacteria Vibrio cholerae. It is transmitted through contaminated food or water and can cause severe dehydration and electrolyte imbalances, which can lead to shock and organ failure if left untreated.   The onset of symptoms is typically sudden and can lead to rapid dehydration and electrolyte imbalances. In severe cases, patients can develop sunken eyes, hoarse voice, decreased urine output, and low blood pressure.  

Toxic food-borne infection (option B) and salmonellosis (option C) can both cause symptoms of acute diarrhea and vomiting, but they are not typically associated with the severe dehydration and electrolyte imbalances seen in cholera.   Dysentery (option D) is a bacterial infection of the intestines that can cause severe diarrhea with blood and mucus. While dysentery can cause dehydration and electrolyte imbalances, the symptoms described in the question are more characteristic of cholera.  

Typhoid fever (option E) is a bacterial infection caused by Salmonella typhi and can cause symptoms such as fever, headache, abdominal pain, and diarrhea. However, the symptoms described in the question, including the acute onset of watery diarrhea, vomiting, and severe dehydration, are not typical of typhoid fever.  

Therefore, based on the patient’s symptoms and clinical presentation, the most likely diagnosis is cholera. Prompt treatment with fluid and electrolyte replacement is essential to prevent complications and reduce mortality.


58. A 28 y.o. man fell seriously ill, he feels chill, has got a fever, body temperature raised up to 38, 50, paroxysmal pain in the left iliac region, frequent defecation in form of fluid bloody and mucous mass. Abdomen palpation reveals painfulness in its left half, sigmoid colon is spasmed. What is the most probable diagnosis?

A. Acute dysentery

B. Amebiasis

C. Colibacillosis

D. Nonspecific ulcerative colitis

E. Malignant tumors of large intestine


Answer: Acute dysentery

Explanation

The patient’s symptoms of fever, chills, paroxysmal pain in the left iliac region, frequent defecation of bloody and mucous mass, and painfulness on palpation of the left half of the abdomen are consistent with acute dysentery.   Acute dysentery is a bacterial infection of the intestines, typically caused by Shigella or other bacteria. It is characterized by symptoms of fever, abdominal pain, and frequent, small-volume stools that are often bloody or mucous.  

Amebiasis (option B) is a parasitic infection caused by the amoeba Entamoeba histolytica. It can cause symptoms of diarrhea, abdominal pain, and fever, but the presence of bloody and mucous stools is less common.   Colibacillosis (option C) is a bacterial infection caused by Escherichia coli (E. coli). It can cause symptoms of diarrhea, abdominal pain, and fever, but the presence of bloody and mucous stools is less common.  

Nonspecific ulcerative colitis (option D) is a chronic inflammatory bowel disease characterized by inflammation and ulcers in the lining of the colon and rectum. While it can cause symptoms of abdominal pain and diarrhea with bloody stools, the acute onset of symptoms described in the question is more characteristic of acute dysentery.  

Malignant tumors of the large intestine (option E) can cause symptoms such as abdominal pain, changes in bowel habits, and rectal bleeding, but the acute onset of symptoms described in the question is not typical of malignancy.   Therefore, based on the patient’s symptoms and clinical presentation, the most likely diagnosis is acute dysentery. Prompt treatment with antibiotics and fluid and electrolyte replacement is essential to prevent complications and reduce morbidity and mortality.


59. The patient was admitted to the hospital on the 7-th day of the disease with complaints of high temperature, headache, pain in the muscles, especially in calf muscles. The dermal integuments and scleras are icteric. There is hemorrhagic rash on the skin. Urine is bloody. The patient went fishing two weeks ago. What is the diagnosis?

A. Leptospirosis

B. Yersiniosis

C. Salmonellosis

D. Brucellosis

E. Trichinellosis


Answer: Leptospirosis

Explanation

The patient’s symptoms of high fever, headache, muscle pain (especially in the calf muscles), icteric skin and sclera, hemorrhagic rash, and bloody urine are consistent with leptospirosis.   Leptospirosis is a bacterial infection caused by spirochetes of the genus Leptospira.

It is typically transmitted through contact with contaminated water or soil, often through recreational activities such as swimming or fishing. The disease can range in severity from mild flu-like symptoms to severe and potentially fatal complications such as kidney failure and bleeding.   Symptoms of leptospirosis typically develop 5-14 days after exposure to the bacteria and can include fever, headache, muscle pain (especially in the calf muscles), vomiting, diarrhea, and skin rash.

In severe cases, patients can develop jaundice, hemorrhage, and kidney failure.   Yersiniosis (option B) is a bacterial infection caused by Yersinia bacteria. It can cause symptoms of fever, abdominal pain, and diarrhea, but is not typically associated with the hemorrhagic rash or muscle pain described in the question.   Salmonellosis (option C) is a bacterial infection caused by Salmonella bacteria. It can cause symptoms of fever, abdominal pain, and diarrhea, but is not typically associated with the hemorrhagic rash or muscle pain described in the question.  

Brucellosis (option D) is a bacterial infection caused by Brucella bacteria. It can cause symptoms such as fever, headache, and joint pain, but is not typically associated with the hemorrhagic rash or bloody urine described in the question.   Trichinellosis (option E) is a parasitic infection caused by eating undercooked or raw meat infected with Trichinella larvae.

It can cause muscle pain and fever, but is not typically associated with the hemorrhagic rash or icteric skin and sclera described in the question.   Therefore, based on the patient’s symptoms and history of exposure to contaminated water while fishing, the most likely diagnosis is leptospirosis. Prompt treatment with antibiotics and supportive care is essential to prevent complications and reduce morbidity and mortality.


60. A 43 y.o. patient was admitted to the hospital with complaints of high temperature of the body and severe headache. On examination: carbuncle is revealed on the forearm. There are intense edema around it, insignificant pain, regional lymphadenitis. The patient is a worker of cattle-ranch. What disease is it necessary to think about first?

A. Anthrax

B. Carcinoma of skin

C. Erysipelas

D. Erysipeloid

E. Eczema


Answer: Anthrax

Explanation

The patient’s symptoms of high fever, severe headache, and the presence of a carbuncle on the forearm with surrounding edema, pain, and regional lymphadenitis are consistent with cutaneous anthrax.  

Anthrax is a bacterial infection caused by Bacillus anthracis and is most commonly acquired through contact with infected animals or animal products, such as wool or hides. Cutaneous anthrax occurs when the bacteria enter the skin through a cut or abrasion and can cause the formation of a carbuncle or black eschar, regional lymphadenitis, and systemic symptoms such as fever and headache.  

Carcinoma of the skin (option B) is a type of skin cancer and is not typically associated with the symptoms described in the question.   Erysipelas (option C) is a bacterial infection of the skin and subcutaneous tissues that can cause symptoms such as fever, chills, and red, swollen skin.

However, the presence of a carbuncle and regional lymphadenitis is not typical of erysipelas.   Erysipeloid (option D) is a bacterial infection of the skin caused by Erysipelothrix rhusiopathiae. It can cause symptoms such as red, swollen skin and fever, but is not typically associated with the formation of a carbuncle or regional lymphadenitis.  

Eczema (option E) is a chronic skin condition characterized by itching, redness, and inflammation of the skin. It is not typically associated with the formation of a carbuncle or systemic symptoms such as fever and headache.   Therefore, based on the patient’s symptoms and history of contact with animals as a cattle-ranch worker, the most likely diagnosis is cutaneous anthrax. Prompt treatment with antibiotics and supportive care is essential to prevent complications and reduce morbidity and mortality.


61. A 16 y.o. female presents with abdominal pain and purpuric spots on the skin. Laboratory investigations reveals a normal platelet count, with haematuria and proteinuria.The most likely diagnosis:

A. Schonlein-Henoch purpura

B. Haemolytic uraemic syndrome

C. Thrombotic thrombocytopenic purpura

D. Heavy metal poisoning

E. Sub acute bacterial endocarditis


Answer:  Schonlein-Henoch purpura

Explanation

The patient’s symptoms of abdominal pain and purpuric spots on the skin, along with laboratory findings of haematuria and proteinuria, are characteristic of Schonlein-Henoch purpura (also known as IgA vasculitis).  

Schonlein-Henoch purpura is a type of vasculitis that affects small blood vessels, typically in the skin, gastrointestinal tract, and kidneys. It is most commonly seen in children and young adults and is often preceded by an upper respiratory tract infection.   Symptoms of Schonlein-Henoch purpura can include abdominal pain, joint pain, purpuric rash (small red or purple spots on the skin), and hematuria (blood in the urine) with or without proteinuria (protein in the urine).

Platelet counts are typically normal in Schonlein-Henoch purpura, which distinguishes it from other conditions that can cause purpura, such as thrombotic thrombocytopenic purpura (TTP).   Haemolytic uraemic syndrome (option B) is a condition characterized by hemolytic anemia, thrombocytopenia (low platelet count), and acute kidney injury.

While it can cause purpura, the normal platelet count in the patient described in the question makes HUS less likely.   Thrombotic thrombocytopenic purpura (option C) is a rare blood disorder characterized by microvascular thrombosis, leading to low platelet counts, hemolytic anemia, and organ damage. It can cause purpura, but the normal platelet count in the patient described in the question makes TTP less likely.  

Heavy metal poisoning (option D) can cause a variety of symptoms, but the combination of abdominal pain and purpuric spots on the skin with haematuria and proteinuria is more suggestive of a vasculitis such as Schonlein-Henoch purpura.  

Subacute bacterial endocarditis (option E) is a bacterial infection of the heart valves and can cause a variety of symptoms including fever, fatigue, and heart murmur. While it can cause purpura, the abdominal pain and laboratory findings of haematuria and proteinuria are not typical of subacute bacterial endocarditis.  

Therefore, based on the patient’s symptoms and laboratory findings, the most likely diagnosis is Schonlein-Henoch purpura. Treatment typically involves supportive care and management of symptoms, although severe cases may require immunosuppressive therapy.


62. A 60 y.o. asthmatic man comes for a check up and complains that he is having some difficulty in “starting to urinate”. Physical examination indicates that the man has blood pressure of 160/100 mm Hg, and a slight enlarged prostate. Which of the following medications would be useful in treating both of these conditions:

A. Doxazosin

B. Labetalol

C. Phetolamine

D. Propranolol

E. Isoproterenol


Answer: Doxazosin

Explanation

Doxazosin is an alpha-blocker medication that is commonly used to treat high blood pressure (hypertension) and symptoms of an enlarged prostate (benign prostatic hyperplasia or BPH). It works by relaxing the muscles in the prostate gland and in blood vessels, which can improve urine flow and lower blood pressure.  

In the case of the asthmatic man with difficulty starting to urinate and an enlarged prostate, doxazosin would be a useful medication to treat both conditions. By relaxing the muscles in the prostate gland, doxazosin can help to improve urine flow and relieve symptoms of BPH. By relaxing blood vessels, it can also help to lower blood pressure and reduce the risk of complications associated with hypertension.   Labetalol (option B) is a beta-blocker medication that is used to treat high blood pressure and certain types of heart disease.

It is not typically used to treat symptoms of an enlarged prostate.   Phentolamine (option C) is an alpha-blocker medication that is used to treat high blood pressure and certain types of heart disease. It is not typically used to treat symptoms of an enlarged prostate.   Propranolol (option D) is a beta-blocker medication that is used to treat high blood pressure, certain types of heart disease, and anxiety. It is not typically used to treat symptoms of an enlarged prostate.  

Isoproterenol (option E) is a beta-agonist medication that is used to treat certain types of heart disease and asthma. It is not typically used to treat symptoms of an enlarged prostate or high blood pressure.  

Therefore, based on the patient’s symptoms and medical history, doxazosin would be the most appropriate medication to treat both his difficulty in starting to urinate and his high blood pressure. However, the patient should be evaluated by a healthcare professional and the appropriate medication and dosage should be determined based on his individual needs and medical history.


63. A 30 y.o. man presents with a history of recurrent pneumonias and a chronic cough production of foul smelling, pirulent sputum, ocassionally glood tinged, which is worse in the morning and on lying down. On physical examination, the patient appears chronically ill with clubbing of fingers, inspiratory rales at the base of lungs posteriorly. Most likely diagnosis:

A. Bronchoectasis

B. Chronic bronchitis

C. Disseminated pulmonary tuberculosis

D. Pulmonary neoplasm

E. Chronic obstructive emphysema


Answer: Bronchoectasis

Explanation

The patient’s symptoms of recurrent pneumonias, chronic cough with foul-smelling, purulent sputum, occasionally blood-tinged, and worse in the morning and on lying down, along with physical examination findings of clubbing of fingers and inspiratory rales at the base of lungs posteriorly, are consistent with a diagnosis of bronchiectasis.  

Bronchiectasis is a chronic respiratory condition characterized by irreversible dilation and damage to the bronchial tubes. It can cause a chronic cough with production of foul-smelling, purulent sputum, recurrent infections, and respiratory failure in severe cases.

Physical examination findings can include clubbing of the fingers and inspiratory rales at the base of lungs posteriorly.   Chronic bronchitis (option B) is a type of chronic obstructive pulmonary disease (COPD) characterized by chronic cough and sputum production. It can cause recurrent infections, but is not typically associated with the foul-smelling, purulent sputum or clubbing of the fingers seen in bronchiectasis.  

Disseminated pulmonary tuberculosis (option C) can cause recurrent infections and respiratory symptoms, but is not typically associated with foul-smelling, purulent sputum or clubbing of the fingers seen in bronchiectasis.   Pulmonary neoplasm (option D) is a type of lung cancer that can cause respiratory symptoms, but is typically associated with other clinical features such as weight loss, fatigue, and hemoptysis (coughing up blood).  

Chronic obstructive emphysema (option E) is a type of COPD characterized by destruction of lung tissue, which can cause shortness of breath, cough, and sputum production. It is not typically associated with the foul-smelling, purulent sputum or clubbing of the fingers seen in bronchiectasis.  

Therefore, based on the patient’s symptoms and physical examination findings, the most likely diagnosis is bronchiectasis. Treatment typically involves management of symptoms, including antibiotics for recurrent infections, bronchodilators to improve airflow, and airway clearance techniques to help remove mucus from the lungs. In severe cases, surgery may be necessary to remove damaged lung tissue.


64. The family doctor examined a patient and diagnosed an acute bleeding of an intestine. What is the doctor’s professional tactics in this situation?

A. The urgent hospitalization in to the surgical department

B. Aminocapronic acid intravenously

C. The urgent hospitalization in to tthe herapeutic department

D. Treatment at a day time hospital

E. Treatment at home


Answer: The urgent hospitalization in to the surgical department

Explanation

Acute bleeding from the intestine is a medical emergency that requires urgent evaluation and management. The most appropriate professional tactic in this situation is to urgently hospitalize the patient into the surgical department for further evaluation and treatment.  

The cause of the bleeding needs to be identified and managed promptly to prevent further complications and potential mortality. Depending on the location and severity of the bleeding, the patient may require endoscopy, surgery, or other interventions to stop the bleeding.   Aminocaproic acid (option B) is a medication that can be used to treat bleeding disorders, but it is not appropriate for the management of acute bleeding from the intestine.  

Urgent hospitalization into the therapeutic department (option C) may be appropriate in some cases, but in the case of acute bleeding from the intestine, urgent evaluation and management in the surgical department is required.  

Treatment at a day time hospital (option D) or at home (option E) is not appropriate for the management of acute bleeding from the intestine, as the patient requires urgent evaluation and treatment in a hospital setting.   Therefore, based on the patient’s diagnosis of acute bleeding from the intestine, the most appropriate professional tactics for the family doctor is to urgently hospitalize the patient into the surgical department for further evaluation and management.


65. A 19 y.o. girl was admitted to Emergency Department: unconsionsness state, cyanosis, myotic pupil, superficial breathing – 12/min. BP- 90/60 mm Hg, Ps78/min. Choose the action corresponding to this clinical situation:

A. Controlled respiration

B. Gastric lavage

C. Oxygen inhalation

D. Caffeine injection

E. Cordiamine injection

Answer: Controlled respiration

Explanation

The patient’s clinical presentation suggests a possible opioid overdose, which can cause respiratory depression, decreased level of consciousness, and constricted (miosis) pupils. The first and most important action in this clinical situation is to ensure adequate oxygenation and ventilation.

Therefore, the appropriate action corresponding to this clinical situation is controlled respiration.   Controlled respiration involves ensuring an open airway and providing assisted ventilation as needed to maintain adequate oxygenation and ventilation. This may involve using a bag-valve-mask device or other respiratory support methods.  

Gastric lavage (option B) is not appropriate in this clinical situation, as it will not address the patient’s respiratory depression and may cause further complications.   Oxygen inhalation (option C) is an important supportive measure in this clinical situation, but it is not the first and most important action.

Controlled respiration should be initiated first, and oxygen inhalation can be provided as needed to maintain adequate oxygenation.   Caffeine injection (option D) and cordiamine injection (option E) are not appropriate in this clinical situation, as they will not address the underlying respiratory depression and may cause further complications.  

Therefore, based on the patient’s clinical presentation, the appropriate action corresponding to this clinical situation is controlled respiration to ensure adequate oxygenation and ventilation. After ensuring adequate ventilation, the patient should be evaluated for possible opioid overdose and appropriate treatment initiated.

66. A 58 y.o. man complaines of severe inspiratory dyspnea and expectoration of frothy and blood-tinged sputum. He has been suffering from essential hypertension and ischemic heart disease. On examination: acrocyanosis, “bubbling”breathing, Ps- 30/min, BP- 230/130 mm Hg, bilateral rales. Choose medicines for treatment.

A. Morphine, furosemide, nitroprusside sodium

B. Theophylline, prednisolon

C. Albuterol, atropine, papaverine

D. Strophanthine, potassium chloride, plathyphylline

E. Cordiamine, isoproterenol

Answer: Morphine, furosemide, nitroprusside sodium


Explanation

The patient’s symptoms and physical examination findings suggest acute pulmonary edema, a medical emergency that requires prompt treatment. Acute pulmonary edema can be caused by heart failure, which may be related to the patient’s history of essential hypertension and ischemic heart disease.   The appropriate medications to treat acute pulmonary edema and manage the underlying heart failure in this patient are morphine, furosemide, and nitroprusside sodium.  

Morphine is a potent analgesic that can also act as a vasodilator, which can reduce the patient’s anxiety and preload on the heart, improving cardiac output and reducing pulmonary congestion.   Furosemide is a loop diuretic that can help to reduce fluid overload and pulmonary congestion by increasing urine output and decreasing fluid accumulation in the lungs.   Nitroprusside sodium is a potent vasodilator that can lower blood pressure and reduce afterload on the heart, thereby improving cardiac output and reducing pulmonary congestion.  

Theophylline and prednisolone (option B) are not appropriate in this clinical situation, as they are not effective in the management of acute pulmonary edema.   Albuterol, atropine, and papaverine (option C) are not appropriate in this clinical situation, as they are primarily used to treat respiratory conditions such as asthma and are not effective in the management of acute pulmonary edema.  

Strophanthine, potassium chloride, and platiphylline (option D) are not appropriate in this clinical situation, as they are not effective in the management of acute pulmonary edema and may even worsen the patient’s condition.   Cordiamine and isoproterenol (option E) are also not appropriate in this clinical situation, as they are not effective in the management of acute pulmonary edema and may even worsen the patient’s condition.  

Therefore, based on the patient’s symptoms and physical examination findings, the appropriate medications for treatment are morphine, furosemide, and nitroprusside sodium to manage acute pulmonary edema and underlying heart failure. The patient should be closely monitored in a hospital setting and further management should be tailored to the individual patient’s needs.


67. A 41 y.o. woman complains of weakness, fatigue, fever up to 380C, rash on the face skin, pain in the wrists and the elbows. On physical examination: erythematous rash on the cheeks with “butterfly”look, the wrists and elbow joints are involved symmetrically, swollen, sensitive, friction rub over the lungs, the heart sounds are weak, regular, HR88/min, BP- 160/95 mm Hg. Hematology shows anemia, leucopenia, lymphopenia; on urinalysis: proteinuria, leukocyturia, casts. What is the main mechanism of disease development?

A. Production of antibodies to doublestranded DNA

B. Production of myocytes antibodies

C. Production of antibodies to endothelial cells

D. Production of myosin antibodies

E. Production of antimitochondrial antibodies


Answer: Production of antibodies to doublestranded DNA

Explanation

The patient’s symptoms and physical examination findings are consistent with systemic lupus erythematosus (SLE), an autoimmune disease that can affect multiple organ systems. SLE is characterized by the production of autoantibodies, including antibodies to double-stranded DNA (dsDNA), which can lead to tissue damage and inflammation.  

The patient’s symptoms and physical examination findings are typical of SLE, including the presence of a characteristic “butterfly” rash on the face, joint pain and swelling, and chest pain with a friction rub over the lungs and weak heart sounds. The patient’s laboratory findings are also consistent with SLE, including anemia, leukopenia, lymphopenia, proteinuria, leukocyturia, and casts in the urine.   Antibodies to dsDNA are a hallmark of SLE and are present in up to 70% of patients with the disease.

These antibodies can cause tissue damage and inflammation by activating the complement system and promoting the deposition of immune complexes in tissues.   Myocytes antibodies (option B), antibodies to endothelial cells (option C), myosin antibodies (option D), and antimitochondrial antibodies (option E) are not typically associated with SLE.  

Therefore, based on the patient’s symptoms, physical examination findings, and laboratory results, the main mechanism of disease development is the production of antibodies to double-stranded DNA, which is characteristic of systemic lupus erythematosus. Treatment typically involves the use of immunosuppressive medications and management of symptoms. The patient should be referred to a rheumatologist for further evaluation and management.


68. A 56 y.o. woman has an acute onset of fever up to 390C with chills, cough, and pain on respiration in the right side of her chest. On physical examination: HR90/min, BP- 95/60 mm Hg, RR- 26/min. There is dullness over the right lung on percussion. On X-ray: infiltrate in the right middle lobe of the lung. What is the diagnosis?

A. Community-acquired lobar pneumonia with moderate severity

B. Community-acquired bronchopneumonia

C. Acute pleurisy

D. Acute lung abscess

E. Hospital-acquired lobar pneumonia


Answer:  Community-acquired lobar pneumonia with moderate severity

Explanation

The patient’s symptoms and physical examination findings suggest an acute bacterial infection of the lung, likely pneumonia. The infiltrate on X-ray confirms the presence of an acute inflammatory process in the right middle lobe of the lung.   Community-acquired lobar pneumonia is a common type of pneumonia that is caused by a variety of bacteria, including Streptococcus pneumoniae, Haemophilus influenzae, and Klebsiella pneumoniae.

The symptoms typically include fever, cough, chest pain, and shortness of breath. The presence of dullness on percussion and an infiltrate on X-ray are consistent with this diagnosis.   Community-acquired bronchopneumonia (option B) is another type of pneumonia that typically involves multiple areas of the lung and is associated with a more gradual onset of symptoms.   Acute pleurisy (option C) is an inflammation of the lining of the lungs and chest wall that typically presents with chest pain that worsens with breathing.

However, the presence of an infiltrate on X-ray and fever are not consistent with this diagnosis.   Acute lung abscess (option D) is a localized collection of pus within the lung parenchyma. While it may present with fever and cough, the presence of an infiltrate on X-ray and dullness on percussion suggest an acute inflammatory process involving the lung tissue rather than a localized abscess.   Hospital-acquired lobar pneumonia (option E) is a type of pneumonia that is acquired in the hospital setting and is typically associated with more resistant bacteria.

The patient’s history does not suggest a hospital-acquired infection.   Therefore, based on the patient’s symptoms, physical examination findings, and X-ray results, the most likely diagnosis is community-acquired lobar pneumonia with moderate severity. Treatment typically involves antibiotics and supportive care, and the patient should be closely monitored in a hospital setting.


69. A 50 y.o. woman for 1 year complained of attacks of right subcostal pain after fatty meal. Last week the attacks have repeated every day and become more painful. What diagnostic study would you recommend?

A. Ultrasound examination of the gallbladder

B. Liver function tests

C. X-ray examination of the gastrointestinal tract

D. Ultrasound study of the pancreas

E. Blood cell count


Answer: Ultrasound examination of the gallbladder

Explanation

The patient’s symptoms are suggestive of biliary colic, which is a type of pain that occurs when a gallstone temporarily blocks the bile duct. Biliary colic typically presents as right subcostal pain that occurs after eating a fatty meal. The fact that the attacks have become more frequent and more painful suggests that there may be a more significant problem with the gallbladder or bile ducts.  

Ultrasound examination of the gallbladder is a non-invasive and effective diagnostic study to evaluate for the presence of gallstones, gallbladder inflammation (cholecystitis), or other abnormalities that may be causing the patient’s symptoms. The test is widely available, safe, and relatively inexpensive.  

Liver function tests (option B) may be useful in evaluating liver function and detecting liver disease, but they are not specific to biliary colic and may not provide a definitive diagnosis.   X-ray examination of the gastrointestinal tract (option C) is not typically used in the evaluation of biliary colic, as it does not provide detailed information about the gallbladder or bile ducts.   Ultrasound study of the pancreas (option D) may be useful in evaluating pancreatic disease, but it is not typically used in the evaluation of biliary colic.  

Blood cell count (option E) may be useful in evaluating for infection or inflammation, but it is not specific to biliary colic and may not provide a definitive diagnosis.   Therefore, based on the patient’s symptoms and history, the most appropriate diagnostic study to recommend is an ultrasound examination of the gallbladder to evaluate for the presence of gallstones or other abnormalities that may be causing the patient’s symptoms.


70. A 25 y.o. woman complained of fatigue, hair loss and brittle nails. The examination revealed pallor of skin, Ps- 94/min, BP- 110/70 mm Hg. On blood count: Hb90 g/L, RBC- 3, 5·1012/L, C.I.- 0,7; ESR- 20 mm/h. Serum iron level was 8,7 mcmol/l. What treatment would you initiate?

A. Ferrous sulfate orally

B. Iron dextrin injections

C. Vitamin B12 intramuscularly

D. Blood transfusion

E. Packed RBCs transfusion


Answer: Ferrous sulfate orally

Explanation

The patient’s symptoms and laboratory results suggest iron deficiency anemia, which is a type of anemia that occurs due to a deficiency in iron, a necessary component for the production of hemoglobin in red blood cells.

The patient’s low hemoglobin level, low serum iron level, and microcytic hypochromic red blood cells on the blood count are supportive of this diagnosis.   The first-line treatment for iron deficiency anemia is oral iron supplementation with ferrous sulfate or another form of iron supplement. Ferrous sulfate is typically dosed at 325 mg three times daily, preferably on an empty stomach with vitamin C to enhance absorption.  

Iron dextrin injections (option B) may be used in cases where oral iron supplementation is not effective or not tolerated, but they are associated with a higher risk of adverse effects and are typically reserved for cases where oral iron is not an option.   Vitamin B12 intramuscularly (option C) is typically used in the treatment of pernicious anemia, which is a type of anemia caused by a deficiency of vitamin B12, but it is not indicated in the treatment of iron deficiency anemia.  

Blood transfusion (option D) or packed red blood cell transfusion (option E) may be indicated in severe cases of anemia with hemodynamic instability or in cases where iron supplementation is not effective, but they are typically reserved for more severe cases of anemia and are not indicated as first-line treatment for iron deficiency anemia.  

Therefore, based on the patient’s symptoms and laboratory results, the most appropriate treatment would be ferrous sulfate supplementation orally. The patient should be advised to take the medication as directed and to follow up with their healthcare provider for repeat laboratory testing to monitor response to treatment.


71. A 38 y.o. woman is seriously ill. She complains of frequent paroxysms of expiratory dyspnea. The last paroxysm lasted over 12 hours and failed to respond to theophylline. The skin is palish gray, moist, RR of 26/min. On auscultation, breath sounds are absent over some areas. Your preliminary diagnosis?

A. Bronchial asthma, status asthmaticus

B. Chronic obstructive bronchitis

C. Atopic bronchial asthma, respiratory failure of the III degree

D. Bronchiectasis, respiratory failure of the II-III degree

E. Ischemic heart disease, pulmonary edema


Answer: Bronchial asthma, status asthmaticus

Explanation

The patient presents with frequent paroxysms of expiratory dyspnea, which is a classic symptom of asthma. The fact that the last paroxysm lasted over 12 hours and did not respond to theophylline suggests that the patient is experiencing a severe exacerbation of asthma, known as status asthmaticus.

The absence of breath sounds over some areas on auscultation may indicate that the patient is experiencing airway obstruction, which is another hallmark of asthma.   Chronic obstructive bronchitis (B) is a chronic condition that is characterized by cough and sputum production, but it typically does not present with paroxysms of dyspnea like asthma.

Atopic bronchial asthma, respiratory failure of the III degree (C) is not a recognized diagnosis, and the term “respiratory failure of the III degree” is not commonly used in medical practice. Bronchiectasis, respiratory failure of the II-III degree (D) is a condition characterized by the widening of the airways, which can lead to chronic cough, sputum production, and recurrent infections, but it is not typically associated with paroxysms of dyspnea.

Ischemic heart disease, pulmonary edema (E) may present with dyspnea, but it typically does not present with the characteristic paroxysms of expiratory dyspnea seen in asthma.   Therefore, based on the presenting symptoms and clinical findings, the most likely diagnosis is bronchial asthma, status asthmaticus.


72. A 19 y.o. girl admitted to the hospital complained of pain in the knee and fever of 38, 60C. She is ill for 2 weeks after acute tonsillitis. On exam, hyperemia and swelling of both knees, temperature is 37, 40C, HR- 94/min, BP- 120/80 mm Hg, and heart border is displaced to the left; S1 is weak, systolic murmur is present. Total blood count shows the following: Hb- 120 g/L, WBC- 9, 8 · 109/L, ESR of 30 mm/L. ECG findings: the rhythm is regular, PQ = 0,24 sec. What is a causative agent of the disease?

A. Beta-hemolytic streptococci

B. Viral-bacterial association

C. Autoimmune disorder

D. Staphylococci

E. Ricchetsia


Answer:  Beta-hemolytic streptococci

Explanation

The patient presents with pain in the knee and fever, which are suggestive of an acute inflammatory process. The hyperemia and swelling of both knees suggest that the patient may be suffering from an arthritis, which is an inflammation of the joints.

The fact that the patient has a history of acute tonsillitis suggests that the arthritis may be a consequence of a post-streptococcal infection, which is a common cause of reactive arthritis.   The presence of a systolic murmur and displaced heart border to the left suggest that the patient may be suffering from rheumatic fever, which is a complication of untreated streptococcal infection. The weak S1 sound may indicate that the patient is experiencing mitral regurgitation, which is a common complication of rheumatic fever.  

The total blood count shows a slightly elevated WBC count and an elevated ESR, which are both indicative of an inflammatory process. The ECG findings are within normal limits, but they do not provide any specific clues to the underlying cause of the patient’s symptoms.  

Therefore, based on the patient’s clinical presentation, history, and laboratory findings, the most likely causative agent of the disease is beta-hemolytic streptococci, which can cause post-streptococcal reactive arthritis and rheumatic fever.


73. A 42 y.o. woman complains of dyspnea, edema of the legs, and tachycardia during small physical exertion. Heart borders are displaced to the left and S1 is accentuated, there is diastolic murmur on apex. The liver is enlarged by 5 cm. What is the cause of heart failure?

A. Mitral stenosis

B. Mitral regurgitation

C. Tricuspid stenosis

D. Tricuspid regurgitation

E. Aortic stenosis

=


Answer:  Mitral stenosis

Explanation

The patient presents with dyspnea, edema of the legs, and tachycardia during small physical exertion, which are all classic symptoms of heart failure. The displacement of the heart borders to the left, accentuated S1, and diastolic murmur on the apex suggest that the patient may be suffering from mitral stenosis, which is a condition in which the opening of the mitral valve is narrowed, leading to impaired blood flow from the left atrium to the left ventricle.   The enlargement of the liver may be due to the back-up of blood that occurs in heart failure, which can lead to congestion in the liver.  

Mitral regurgitation (B) would typically present with a systolic murmur, rather than a diastolic murmur, and would not necessarily lead to an enlargement of the liver. Tricuspid stenosis (C) is a rare condition, and it typically presents with symptoms that are similar to those of mitral stenosis.

Tricuspid regurgitation (D) would typically present with a systolic murmur, rather than a diastolic murmur, and would not necessarily lead to an enlargement of the liver. Aortic stenosis (E) would typically present with symptoms that are different from those of mitral stenosis, such as syncope, angina, and exertional dyspnea.   Therefore, based on the patient’s clinical presentation and physical exam findings, the most likely cause of heart failure is mitral stenosis.


74. A 54 y.o. woman complains of increasing fatigue and easy bruising of 3 weeks’ duration. Physical findings included pale, scattered ecchymoses and petechiae and mild hepatosplenomegaly. In blood: RBC2, 5 · 1012/L; Hb- 73 g/L; HCT- 20%; PLT23·109/L; and WBC- 162·109/L with 82% blasts, that contained Auric rods; peroxidase stain was positive. What is the most probable diagnosis?

A. Acute leukemia

B. Chronic leukemia

C. Thrombocytopenia

D. Hemolytic anemia

E. Megaloblastic anemia


Answer: Acute leukemia

Explanation

The patient presents with increasing fatigue and easy bruising, which are common symptoms of hematological malignancies such as leukemia. The presence of scattered ecchymoses and petechiae suggests that the patient is experiencing bleeding due to a low platelet count. The mild hepatosplenomegaly may be due to infiltration of leukemic cells in these organs.   The blood test results show a low RBC count, low hemoglobin, and low hematocrit, which are indicative of anemia.

The low platelet count and high white blood cell count with 82% blasts suggest that the patient is experiencing a hematological malignancy, most likely acute leukemia.   The presence of Auric rods and positive peroxidase stain on the blasts are characteristic of acute myeloid leukemia (AML), which is a type of acute leukemia that arises from myeloid progenitor cells. However, the diagnosis of acute leukemia requires further confirmation with additional diagnostic tests, such as bone marrow aspiration and biopsy.  

Chronic leukemia (B) typically presents with a slower onset of symptoms and a more gradual increase in the number of abnormal white blood cells. Thrombocytopenia (C) can cause similar symptoms to those seen in this patient, but it would not explain the presence of blasts in the blood. Hemolytic anemia (D) typically presents with a different set of symptoms, such as jaundice, and would not explain the presence of blasts in the blood.

Megaloblastic anemia (E) is typically caused by a deficiency in vitamin B12 or folate, and would not explain the presence of blasts in the blood.   Therefore, based on the patient’s clinical presentation and blood test results, the most probable diagnosis is acute leukemia, specifically acute myeloid leukemia.


75. A 39 y.o. woman complaines of squeezed epigastric pain 1 hour after meal and heartburn. She had been ill for 2 years. On palpation, there was moderate tenderness in pyloroduodenal area. Antral gastritis was revealed on gastroscopy. What study can establish genesis of the disease?

A. Revealing of Helicobacter infection in gastric mucosa

B. Detection of autoantibodies in the serum

C. Gastrin level in blood

D. Examination of stomach secretion

E. Examination of stomach motor function


Answer: Revealing of Helicobacter infection in gastric mucosa

Explanation

The patient presents with squeezed epigastric pain and heartburn, which are common symptoms of gastritis. The moderate tenderness in the pyloroduodenal area suggests that the patient may be suffering from an inflammation of the stomach lining in that area, known as antral gastritis.  

The most common cause of antral gastritis is Helicobacter pylori infection, which is a bacterial infection that can cause inflammation of the stomach lining. Therefore, the most appropriate study to establish the genesis of the disease is to reveal the presence of Helicobacter pylori infection in the gastric mucosa.  

This can be done through a variety of tests, including a rapid urease test, histological examination of biopsy specimens, serologic testing for H. pylori antibodies, or breath tests. The choice of test may depend on the patient’s clinical presentation and the availability of testing facilities.  

Detection of autoantibodies in the serum (B) may be useful in the diagnosis of autoimmune gastritis, but it is not typically used to diagnose Helicobacter pylori infection. Gastrin level in blood (C) may be useful in the diagnosis of Zollinger-Ellison syndrome, which is a rare condition characterized by the overproduction of gastrin, but it is not typically used to diagnose Helicobacter pylori infection.

Examination of stomach secretion (D) and examination of stomach motor function (E) may be useful in the evaluation of other conditions, such as peptic ulcer disease or gastroparesis, but they are not typically used to diagnose Helicobacter pylori infection.  

Therefore, based on the patient’s clinical presentation and the most likely cause of antral gastritis, the most appropriate study to establish the genesis of the disease is revealing of Helicobacter pylori infection in gastric mucosa.


76. A male, 50 y.o., has a black flat mole on the skin of the leg for 10 years. Since 4 months ago the shapes of the mole become irregular. What diagnostic methods should be used?

A. Excision biopsy

B. Fine needle biopsy

C. Smear for microscopic examination

D. Incision biopsy

E. Thermography


Answer:  Excision biopsy

Explanation

The patient presents with a black flat mole on the skin of the leg that has been present for 10 years. However, the shape of the mole has become irregular over the past 4 months, which is concerning for the development of a malignant melanoma.  

Given the possibility of malignant transformation, the most appropriate diagnostic method in this case would be an excision biopsy of the entire mole. This would involve removing the entire mole, along with a small amount of surrounding tissue, and sending it to a pathology laboratory for analysis.  

A fine needle biopsy (B) or a smear for microscopic examination (C) may be useful in the diagnosis of other skin conditions, such as infections or inflammatory disorders, but they would not be appropriate for the diagnosis of a malignant melanoma.   An incision biopsy (D) may be useful in the diagnosis of some skin conditions, but it would not be appropriate for the diagnosis of a malignant melanoma, as it may not provide a representative sample of the entire lesion.  

Thermography (E) is a non-invasive imaging technique that can be used to evaluate blood flow and temperature patterns in the skin, but it is not typically used in the diagnosis of skin lesions such as moles.   Therefore, based on the patient’s clinical presentation and the possibility of malignant transformation, the most appropriate diagnostic method in this case would be an excision biopsy of the entire mole.


77. The highest risk of congenital anomalies probably occurs when human embryos or fetuses are exposed to ionizing radiation. During which part of gestational period does it occur?

A. 18-45 days after conception

B. The first 7 days

C. 10-14 days after conception

D. 90-120 days after conception

E. The third trimester


Answer: 18-45 days after conception

Explanation

The highest risk of congenital anomalies occurring as a result of exposure to ionizing radiation occurs during the period of organogenesis, which takes place between approximately 18 and 45 days after conception in humans. During this period, the developing embryo is particularly vulnerable to the effects of ionizing radiation, as the major organs and organ systems are being formed.  

Exposure to ionizing radiation during this critical period can lead to a wide range of congenital anomalies, including structural malformations, functional abnormalities, growth retardation, and developmental delays. The severity of the effects depends on the dose and timing of the exposure, as well as the individual susceptibility of the developing embryo.  

Exposure to ionizing radiation outside of the period of organogenesis may also have harmful effects on the developing fetus, but the risk of congenital anomalies is generally lower. The effects of ionizing radiation exposure during the third trimester (E) may include growth retardation, cognitive impairment, and an increased risk of cancer, but the risk of congenital anomalies is relatively low during this period.  


Therefore, based on the timing of organogenesis in humans and the known effects of ionizing radiation exposure on fetal development, the highest risk of congenital anomalies occurring as a result of exposure to ionizing radiation occurs during the period of organogenesis, which takes place between approximately 18 and 45 days after conception.


78. A 7 d.o. boy is admitted to the hospital for evaluation of vomiting and dehydration. Physical examination is otherwise normal except for minimal hyperpigmentation of the nipples. Serum sodium and potassium concentrations are 120 meq/L and 9 meq/L respectively. The most likely diagnosis is:

A. Congenital adrenal hyperplasia

B. Pyloric stenosis

C. Secondary hypothyroidism

D. Panhypopituitarism

E. Hyperaldosteronism


Answer:  Congenital adrenal hyperplasia

Explanation

The patient presents with vomiting and dehydration, which may be due to an electrolyte imbalance caused by a hormonal disorder. The minimal hyperpigmentation of the nipples may be a sign of increased melanin production, which is a characteristic feature of congenital adrenal hyperplasia.  

The laboratory test results show low serum sodium and very low serum potassium concentrations, which are consistent with an adrenal insufficiency. Congenital adrenal hyperplasia is a genetic disorder that leads to a deficiency in one of the enzymes involved in the synthesis of cortisol, which can result in an adrenal insufficiency and electrolyte imbalances.  

Pyloric stenosis (B) typically presents with projectile vomiting in infants, but it does not typically cause electrolyte imbalances. Secondary hypothyroidism (C) and panhypopituitarism (D) may cause electrolyte imbalances as a result of hormonal deficiencies, but they would not typically present with hyperpigmentation of the nipples.

Hyperaldosteronism (E) typically presents with hypertension and hypokalemia, rather than hyponatremia and hyperkalemia.   Therefore, based on the patient’s clinical presentation and laboratory test results, the most likely diagnosis is congenital adrenal hyperplasia.


79. A 7 y.o. boy has crampy abdominal pain and a rash on the back of his legs and buttocks as well as on the extensor surfaces of his forearms. Laboratory analysis reveals proteinuria and microhematuria. He is most likely to be affected by:

A. Anaphylactoid purpura

B. Systemic lupus erythematosus

C. Poststreptococcal glomerulonephritis

D. Polyarteritis nodosa

E. Dermatomyositis


Answer:  Anaphylactoid purpura

Explanation

The patient presents with crampy abdominal pain and a rash on the back of his legs and buttocks, as well as on the extensor surfaces of his forearms. These symptoms are characteristic of anaphylactoid purpura, also known as Henoch-Schönlein purpura (HSP).  

HSP is a type of vasculitis that affects small blood vessels, causing inflammation and damage to the vessel walls. The symptoms may include abdominal pain, joint pain, and a rash that typically appears on the lower extremities and buttocks. Proteinuria and microhematuria are common findings in HSP, as the inflammation and damage to the blood vessels can affect the kidneys.   Systemic lupus erythematosus (B) is an autoimmune disorder that can affect multiple organs, including the skin, joints, and kidneys.

However, the rash in SLE typically appears on the face and is not typically associated with abdominal pain. Poststreptococcal glomerulonephritis (C) is a type of kidney inflammation that can occur after a streptococcal infection, but it typically presents with fever and other signs of infection, and the rash is not a common feature.

Polyarteritis nodosa (D) is a rare type of vasculitis that affects medium-sized arteries, and it typically presents with fever, weight loss, and muscle and joint pain. Dermatomyositis (E) is a type of inflammatory myopathy that affects the muscles and skin, but it typically presents with muscle weakness and a characteristic rash on the face and hands.  

Therefore, based on the patient’s clinical presentation and laboratory findings, the most likely diagnosis is anaphylactoid purpura, also known as Henoch-Schönlein purpura.


80. A 27 y.o. patient has been having for almost a year fatigue, hyperhidrosis, heaviness in the left hypochondrium, especially after meals. Objectively: spleen and liver enlargement. In blood: erythrocytes – 3, 2 · 1012/l, Hb- 100 g/l, colour index – 0,87, leukocutes – 100 · 109/l, basophils – 7%, eosinophils – 5%, myelocytes – 15%, juveniles – 16%, stab neutrophils – 10%, segmentonuclear leukocytes – 45%, lymphocytes – 2%, monocytes – 0%, reticulocytes – 0,3%, thrombocytes – 400 · 109/l, ESR- 25 mm/h. What is the most probable diagnosis?

A. Chronic myeloleukosis

B. Chronic lympholeukosis

C. Acute leukosis

D. Erythremia

E. Hepatocirrhosis


Answer: Chronic myeloleukosis

 Explanation

The patient presents with fatigue, hyperhidrosis, and splenomegaly, which are characteristic features of CML. In addition, the laboratory test results show leukocytosis with a left shift, including an increase in myelocytes and juvenile granulocytes, which are also typical of CML.   CML is a type of myeloproliferative neoplasm, which is a group of disorders characterized by the abnormal growth and development of myeloid cells in the bone marrow.

The abnormal cells can accumulate in the bone marrow, spleen, and other organs, leading to symptoms such as fatigue, weight loss, and splenomegaly.   Chronic lymphocytic leukemia (B) is a type of leukemia that affects lymphocytes, rather than myeloid cells, and it typically presents with lymphadenopathy, rather than splenomegaly.

Acute leukemia (C) typically presents with more severe symptoms, such as fever, bleeding, and bone pain, and the laboratory test results would show more pronounced abnormalities, such as anemia and thrombocytopenia.

Erythremia (D), also known as polycythemia vera, is a type of myeloproliferative neoplasm that is characterized by an increase in red blood cells, rather than white blood cells. Hepatocirrhosis (E) may cause liver enlargement, but it would not typically cause splenomegaly or leukocytosis.   Therefore, based on the patient’s clinical presentation and laboratory test results, the most probable diagnosis is chronic myeloid leukemia (CML).


81. A 28 y.o. woman comes to the Emergency Room with a slightly reddened, painful “knot”, 8 cm above the medial malleolus. Examination in the standing position demonstrates a distended vein above and below the mass There are no other abnormalities on physical examination. The most likely diagnosis is:

A. Superficial venous thrombosis

B. Early deep vein thrombosis

C. Insect bite

D. Cellulitis

E. Subcutaneous hematoma


Answer:  Superficial venous thrombosis

Explanation

The patient presents with a painful, reddened “knot” located 8 cm above the medial malleolus, along with a distended vein above and below the mass. These findings are characteristic of superficial venous thrombosis, which is a blood clot that forms in a vein close to the surface of the skin.   Superficial venous thrombosis typically presents with pain, redness, and swelling along the affected vein, and it is often associated with a distended or palpable vein.

The condition is typically benign and self-limiting, but it can be associated with an increased risk of deep vein thrombosis (DVT) and pulmonary embolism (PE), particularly in patients with underlying risk factors.  

Early deep vein thrombosis (B) may present with similar symptoms, but it typically does not cause a palpable or distended vein. Insect bite (C) may cause localized swelling and redness, but it would not typically result in a visible vein or a palpable mass. Cellulitis (D) is a bacterial infection that may cause redness, swelling, and pain, but it would not typically result in a palpable vein or a “knot” in the affected area.

Subcutaneous hematoma (E) may cause localized swelling and tenderness, but it would not typically result in a distended vein or a palpable mass.   Therefore, based on the patient’s clinical presentation and physical examination findings, the most likely diagnosis is superficial venous thrombosis.


82. A patient with a history of coronary artery disease and atrial fibrillation has the onset of sudden pain and weakness of the left leg. Examination reveals a cool, pale extremity with absent pulses below the groin and normal contralateral leg. The most likely diagnosis is:

A. Arterial embolism

B. Arterial thrombosis

C. Acute thrombophlebitis

D. Cerebrovascular accident

E. Dissecting aortic aneurysm


Answer: Arterial embolism

Explanation

The patient presents with sudden pain and weakness of the left leg, along with a cool, pale extremity and absent pulses below the groin. These findings are consistent with an acute ischemic event, such as an arterial embolism.  

Arterial embolism is a blockage of an artery caused by a clot or other material (embolus) that travels from another part of the body, typically the heart or proximal arterial system. Patients with a history of coronary artery disease and atrial fibrillation are at increased risk of developing arterial embolism.  

Arterial thrombosis (B) is a blockage of an artery caused by a clot that forms locally in the affected vessel, rather than traveling from another part of the body. Acute thrombophlebitis (C) is a clot that forms in a vein, rather than an artery, and it typically presents with localized pain, swelling, and redness.

Cerebrovascular accident (D), also known as stroke, typically presents with neurological symptoms, such as weakness, numbness, or difficulty speaking. Dissecting aortic aneurysm (E) typically presents with sudden, severe chest or back pain, which may radiate to the arms or legs.   Therefore, based on the patient’s clinical presentation and medical history, the most likely diagnosis is arterial embolism.


83. A 30 y.o. man complains of sharp pain in the right ear, hearing loss, high temperature for three days. Objectively: right ear whispering language – 0,5 m, external ear is intact, otoscopically – eardrum protrusion, hyperemia and swelling, loss of landmarks. What disease is it?

A. Acute purulent otitis media

B. Acute mastoiditis

C. Chronic secretory otitis media

D. Chronic purulent otitis media

E. Eustachian tube disfunction


Answer: Acute purulent otitis media

Explanation

The patient presents with sharp pain in the right ear, hearing loss, and fever, along with otoscopic findings of eardrum protrusion, hyperemia, and swelling, with loss of landmarks. These findings are consistent with acute purulent otitis media, which is an infection of the middle ear that is typically caused by bacteria.  

Acute purulent otitis media typically presents with symptoms such as ear pain, hearing loss, and fever, and otoscopic findings of a bulging, erythematous eardrum with loss of landmarks. The condition is more common in children, but it can also occur in adults.  

Acute mastoiditis (B) is a complication of acute otitis media that occurs when the infection spreads to the mastoid bone, causing inflammation and swelling. Chronic secretory otitis media (C) is a condition in which fluid accumulates in the middle ear, leading to hearing loss, but it typically does not cause pain or fever. Chronic purulent otitis media (D) is a long-standing infection of the middle ear that is typically associated with a perforated eardrum and drainage of pus from the ear.

Eustachian tube dysfunction (E) is a condition in which the tube that connects the middle ear to the back of the throat becomes blocked, leading to changes in pressure in the middle ear and potential hearing loss, but it typically does not cause pain or fever.   Therefore, based on the patient’s clinical presentation and otoscopic findings, the most likely diagnosis is acute purulent otitis media.


84. A 22 y.o. man complains of acute throat pain, increasing upon swallowing during 3 days. Body temperature 38, 30, neck lymph nodules are slightly enlarged and painful. Pharyngoscopically – tonsilar hyperemia, enlargement and edema, tonsils are covered by round yellow fi- brinous patches around crypts openings. Beta-haemolytic streptococcus in swab analysis. What is the diagnosis?

A. Acute membranous tonsilitis

B. Acute follicular tonsilitis

C. Pharyngeal diphtheria

D. Infectious mononucleosis

E. Pharyngeal candidosis


Answer: Acute membranous tonsilitis

Explanation

The patient presents with acute throat pain, fever, and enlarged and painful neck lymph nodes, along with pharyngoscopic findings of tonsillar hyperemia, enlargement, and edema, with tonsils covered by round yellow fibrinous patches around crypts openings.

The presence of beta-hemolytic streptococcus in the swab analysis suggests a bacterial etiology.   Acute membranous tonsillitis, also known as acute tonsillitis or acute pharyngitis, is an infection of the tonsils that is typically caused by bacteria, such as streptococcus pyogenes. The condition typically presents with symptoms such as sore throat, fever, and enlarged cervical lymph nodes, along with pharyngoscopic findings of tonsillar inflammation and exudate.  

Acute follicular tonsillitis (B) is a variant of acute tonsillitis in which the tonsils are covered by small, yellow-white follicular exudates. Pharyngeal diphtheria (C), caused by Corynebacterium diphtheriae, is a rare but potentially life-threatening infection that typically presents with a grayish-white membrane covering the tonsils and pharynx.

Infectious mononucleosis (D) typically presents with symptoms such as sore throat, fever, and fatigue, along with lymphadenopathy and splenomegaly. Pharyngeal candidiasis (E) typically presents with white patches on the tongue and buccal mucosa, rather than the tonsils.   Therefore, based on the patient’s clinical presentation and pharyngoscopic findings, the most likely diagnosis is acute membranous tonsillitis.


85. A patient is staying in the hospital with the diagnosis of abdominal typhus. During the 3-d week from the beginning of the disease the patient stopped keeping diet and confinement to bed. As a result the body temperature and rapid pulse decreased and melena appeared. What kind of complications should we think about first of all?

A. Intestinal haemorrhage

B. Thrombophlebitis

C. Meningitis

D. Nephroso-nephritis

E. Hepatite


Answer:  Intestinal haemorrhage

Explanation

The patient has been diagnosed with abdominal typhus and has been in the hospital for three weeks. However, during the third week, the patient stopped following the recommended diet and bed rest, and subsequently developed melena (black, tarry stools), along with a decrease in body temperature and rapid pulse. These symptoms suggest the possibility of intestinal bleeding, which can be a serious complication of abdominal typhus.  

Intestinal hemorrhage is a known complication of abdominal typhus, which is a bacterial infection caused by Salmonella typhi. The infection can cause inflammation and ulceration of the intestinal mucosa, which can lead to bleeding and perforation of the intestines.

Patients with abdominal typhus are typically advised to rest and follow a specific diet to reduce the risk of these complications.   Thrombophlebitis (B) is inflammation of a vein that is typically accompanied by the formation of a blood clot.

Meningitis (C) is inflammation of the membranes that surround the brain and spinal cord. Nephroso-nephritis (D) is a form of kidney inflammation. Hepatitis (E) is inflammation of the liver.   Therefore, based on the patient’s clinical presentation and medical history, the most likely complication is intestinal hemorrhage, which is a known complication of abdominal typhus.


86. The patients has sustained blunt trauma to the chest. Which of the following would most likely be the cause of acute cardiopulmonary collapse?

A. Pneumothorax

B. Hemothorax

C. Pulmonary contusion

D. Rib fractures

E. Acute adult respiratory distress syndrome (ARDS)


Answer: Pneumothorax

Explanation

Blunt trauma to the chest can cause a variety of injuries, including pneumothorax, hemothorax, pulmonary contusion, rib fractures, and acute adult respiratory distress syndrome (ARDS). However, pneumothorax is the most likely cause of acute cardiopulmonary collapse.  

Pneumothorax is the presence of air in the pleural space, which can lead to collapse of the lung and compression of the heart and great vessels. In cases of tension pneumothorax, the pressure in the pleural space can increase to the point of causing acute cardiopulmonary collapse, which is a life-threatening emergency.  

Hemothorax (B) is the presence of blood in the pleural space, which can also lead to collapse of the lung and compression of the heart and great vessels. However, it typically does not cause acute cardiopulmonary collapse unless it is associated with other injuries.

Pulmonary contusion (C) is a bruise of the lung tissue that can lead to respiratory failure, but it typically does not cause acute cardiopulmonary collapse. Rib fractures (D) can cause pain and difficulty breathing, but they typically do not cause acute cardiopulmonary collapse unless they are associated with other injuries.

Acute adult respiratory distress syndrome (ARDS) (E) is a severe form of respiratory failure that can develop in response to various injuries or illnesses, but it typically does not cause acute cardiopulmonary collapse.   Therefore, based on the mechanism of injury and the potential complications, the most likely cause of acute cardiopulmonary collapse after blunt trauma to the chest is pneumothorax.


87. A patient has got pain in the axillary area, rise of temperature developed 10 hours ago. On examination: shaky gait is evident, the tongue is coated with white coating. The pulse is frequent. The painful lymphatic nodes are revealed in the axillary area. The skin is erythematous and glistering over the lymphatic nodes. What is the most probable diagnosis?

A. Bubonic plague

B. Acute purulent lymphadenitis

C. Lymphogranulomatosis

D. Anthrax

E. Tularemia


Answer: Bubonic plague

Explanation

Bubonic plague is a bacterial infection caused by Yersinia pestis. It is typically transmitted to humans through the bites of infected fleas or contact with infected animals. The disease can cause fever, chills, headache, muscle aches, and swollen lymph nodes, which are often painful. In severe cases, bubonic plague can progress to septicemia or pneumonia, which can be life-threatening.  

The patient in this case has pain in the axillary area and swollen, painful lymph nodes, which are typical symptoms of bubonic plague. The development of a white-coated tongue and a high pulse rate are also consistent with the disease. The erythematous and glistening skin over the lymph nodes may be due to inflammation or edema caused by the infection.  

Acute purulent lymphadenitis (B) is a bacterial infection of the lymph nodes that typically presents with swollen, painful lymph nodes that may be accompanied by fever and other symptoms. However, the rapid onset of symptoms and the presence of glistening skin over the lymph nodes suggest a more severe infection, such as bubonic plague.  

Lymphogranulomatosis (C), also known as Hodgkin’s lymphoma, is a type of cancer that affects the lymphatic system. It typically presents with painless lymph node enlargement and may be accompanied by other symptoms, such as fever, night sweats, and weight loss.  

Anthrax (D) is a bacterial infection caused by Bacillus anthracis. It can cause skin, lung, or gastrointestinal disease, depending on the route of exposure. However, the symptoms described in this case are not typical of anthrax.  

Tularemia (E) is a bacterial infection caused by Francisella tularensis. It is typically transmitted through contact with infected animals or bites from infected ticks or deer flies. The disease can cause fever, chills, headache, and swollen lymph nodes, but the erythematous and glistening skin over the lymph nodes is not typical of tularemia.   Therefore, based on the patient’s symptoms and examination findings, the most probable diagnosis is bubonic plague.


88. A 56 y.o. patient has worked at the aluminium plant over 20 years. Within 3 last years he has got loosening of teeth, bone and joint pains, piercing pains in heart region, vomiting. The provisional diagnosis is:

A. Fluorine intoxication

B. Mercury intoxication

C. Lead intoxication

D. Phosphorus intoxication

E. Manganese intoxication


Answer: Fluorine intoxication

Explanation

Fluorine is a chemical element that is used in the production of aluminum and other metals. Fluorine intoxication can occur as a result of exposure to high levels of airborne fluorine compounds in industrial settings. The symptoms of fluorine intoxication can include loosening of teeth, bone and joint pains, and gastrointestinal symptoms such as vomiting.  

The patient in this case has worked at an aluminum plant for over 20 years, which puts him at risk for fluorine intoxication. His symptoms, including loosening of teeth, bone and joint pains, and vomiting, are consistent with the disease. The piercing pains in the heart region could also be related to the toxic effects of fluorine on the cardiovascular system.  

Mercury intoxication (B) can occur as a result of exposure to elemental mercury or certain organic forms of mercury. The symptoms of mercury intoxication can include neurological symptoms such as tremors, memory loss, and personality changes, as well as gastrointestinal symptoms and kidney damage. However, the patient’s occupational history and symptoms are not consistent with mercury intoxication.   Lead intoxication (C) can occur as a result of exposure to lead in industrial or environmental settings.

symptoms of lead intoxication can include neurological symptoms, anemia, and gastrointestinal symptoms. However, the patient’s symptoms are not typical of lead intoxication, and his occupational history suggests a different type of exposure.   Phosphorus intoxication (D) can occur as a result of ingestion of large amounts of elemental phosphorus or certain organic forms of phosphorus.

The symptoms of phosphorus intoxication can include gastrointestinal symptoms, neurological symptoms, and bone and joint pain. However, the patient’s occupational history and symptoms are not consistent with phosphorus intoxication.   Manganese intoxication (E) can occur as a result of exposure to high levels of manganese in industrial or environmental settings.

The symptoms of manganese intoxication can include neurological symptoms such as tremors and difficulty walking, as well as psychiatric symptoms and gastrointestinal symptoms. However, the patient’s occupational history and symptoms are not consistent with manganese intoxication.   Therefore, based on the patient’s occupational history and symptoms, the most probable diagnosis is fluorine intoxication.


89. A 34 y.o. patient has been suffering from pulmonary tuberculosis for 7 years; he complains of muscle feebleness, weight loss, diarrheas, increased frequency of urination. Objectively: hyperpigmentation of skin, gums, internal cheek surfaces. AP is 90/58 mm Hg. Blood count: erythrocutes – 3, 1 · 1012/L, Hb- 95 g/L, C.I.- 0,92; leukocytes – 9, 4 · 109/L, eosinophils – 7, segmentonuclear leukocytes – 45, stab neutrophils – 1, lymphocytes – 40, monocytes – 7, Na+- 115 mmole/L, +- 7,3 mmole/L. What is the preliminary di-agnosis?

A. Primary insufficiency of adrenal cortex

B. Pheochromocytoma

C. Primary hyperaldosteronism

D. Congenital adrenal hyperplasia

E. Diabetes insipidus


Answer:  Primary insufficiency of adrenal cortex

Explanation

Primary insufficiency of the adrenal cortex, also known as Addison’s disease, is a condition in which the adrenal glands do not produce enough of the hormones cortisol and aldosterone. It can be caused by autoimmune disease, infections, cancer, or other factors. Symptoms of Addison’s disease can include weakness, fatigue, weight loss, nausea, vomiting, and hyperpigmentation of the skin and mucous membranes.  

The patient in this case has been suffering from pulmonary tuberculosis for 7 years, which puts him at risk for developing secondary adrenal insufficiency. The symptoms of muscle weakness, weight loss, and diarrhea are consistent with adrenal insufficiency, as is the hyperpigmentation of the skin, gums, and internal cheek surfaces. The low blood pressure reading of 90/58 mm Hg is also consistent with adrenal insufficiency.  

The laboratory results show low levels of erythrocytes and hemoglobin, which can be a result of chronic illness and malnutrition. The low sodium level and slightly acidic pH suggest hyponatremia, which can be a result of aldosterone deficiency in adrenal insufficiency.  

Pheochromocytoma (B) is a rare tumor of the adrenal gland that can cause high blood pressure, headaches, and sweating. The patient’s low blood pressure and lack of these symptoms make pheochromocytoma an unlikely diagnosis.   Primary hyperaldosteronism (C) is a condition in which the adrenal gland produces too much aldosterone, which can lead to high blood pressure and low potassium levels.

The patient’s low blood pressure and normal potassium level make primary hyperaldosteronism an unlikely diagnosis.   Congenital adrenal hyperplasia (D) is a group of genetic disorders that affect the adrenal gland’s ability to produce cortisol. The symptoms of congenital adrenal hyperplasia can include growth failure, virilization, and salt wasting. The patient’s symptoms and laboratory results are not consistent with congenital adrenal hyperplasia.  

Diabetes insipidus (E) is a condition in which the body does not produce enough antidiuretic hormone (ADH), which can lead to excessive thirst and urination. The patient’s increased frequency of urination is not consistent with diabetes insipidus, as this condition typically causes large amounts of dilute urine.   Therefore, based on the patient’s symptoms, physical examination findings, and laboratory results, the preliminary diagnosis is primary insufficiency of the adrenal cortex.


90. 3 weeks ago a patient was ill with tonsillitis. Clinical examination reveals edema, arterial hypertension, hematuria, proteinuria (1,8 g/per day), granular and erythrocital casts. What is the preliminary diagnosis?

A. Glomerulonephritis

B. Cystitis

C. Pyelonephritis

D. Intestinal nephritis

E. Renal amyloidosis

Answer:  Glomerulonephritis

Explanation

Glomerulonephritis is a group of diseases that affect the glomeruli, which are the tiny filters in the kidneys that remove waste products from the blood. It can be caused by various factors, including infections, autoimmune disorders, and medications. The symptoms of glomerulonephritis can include edema, hypertension, hematuria, and proteinuria.  

In this case, the patient had tonsillitis three weeks ago, which could have triggered an immune response that led to glomerulonephritis. The presence of arterial hypertension, hematuria, and proteinuria (1.8 g/day) are all consistent with glomerulonephritis. The presence of granular and erythrocyte casts in the urine also suggests glomerular damage.  

Cystitis (B) is an infection of the urinary bladder that typically causes painful urination, urinary frequency, and urgency. It is less likely in this case, as the patient’s symptoms and laboratory results suggest a more systemic condition.   Pyelonephritis (C) is an infection of the kidneys that can cause fever, flank pain, and urinary symptoms.

While it can also cause hematuria and proteinuria, the absence of fever and flank pain make pyelonephritis less likely in this case.   Intestinal nephritis (D) is a rare condition in which the kidneys are affected by inflammation in the intestines. It can cause hematuria and proteinuria, but is less likely in this case as the patient’s symptoms and history suggest a more likely diagnosis of glomerulonephritis.  

Renal amyloidosis (E) is a condition in which abnormal proteins build up in the kidneys, causing damage to the glomeruli and other structures. It can cause hematuria and proteinuria, but is less likely in this case as the patient’s recent tonsillitis suggests an infectious or immune-mediated cause for the symptoms.   Therefore, based on the patient’s symptoms and laboratory results, the preliminary diagnosis is glomerulonephritis.


91. A patient complains of feeling heaviness behind his breast bone, periodical sensation of food stoppage, dysphagy. During the X-ray examination barium contrast revealed a single saccular outpouching of anterodextral esophagus wall with regular contours and rigidly outlined neck. What is the most probable diagnosis?

A. Esophageal diverticulum

B. Cancer of esophagus

C. Hiatal hernia

D. Varix dilatation of esophageal veins

E. Esophageal polyp

Answer: Esophageal diverticulum

Explanation

1. The symptoms described include heaviness in the chest, food impactions, and difficulty swallowing (dysphagia). These are classic symptoms of esophageal diverticulum.   2. The barium study found “a single saccular outpouching of anterodextral esophagus wall with regular contours and rigidly outlined neck.”

This describes the shape and characteristics of an esophageal diverticulum.   3. Hiatal hernia and esophageal cancer would be less likely given the description of a single, saccular pouching with regular contours. Those conditions typically present differently on imaging.   4. Varix dilatation and esophageal polyp would also not typically cause the described symptoms of heaviness and dysphagia.

And they would appear differently on barium study – not as a saccular outpouching.   So in summary, the symptoms, exam findings and imaging characteristics described best match those of an esophageal diverticulum, making that the most probable diagnosis in this case. I hope this explanation helps clarify the reasoning behind selecting answer A! Let me know if you have any other questions.


92. A 40 y.o. patient complains of yellowish discharges from the vagina. Bimanual examination: no pathological changes. The smear contains Trichomonas vaginalis and blended flora. Colposcopy: two hazy fields on the front labium, with a negative Iodum test. Your tactics:

A. Treatment of specific colpitis and with the subsequent biopsy

B. Diathermocoagulation of the cervix of the uterus

C. Specific treatment of Trichomonas colpitis

D. Cervix ectomy

E. Cryolysis of cervix of the uterus


Answer:  Treatment of specific colpitis and with the subsequent biopsy

Explanation

The key details in the information provided:   – The patient is a 40 year old woman complaining of yellowish vaginal discharge – On bimanual exam, there are no pathological changes noted – The vaginal smear shows Trichomonas vaginalis present, along with a mixed vaginal flora – Colposcopy reveals two hazy fields on the labia, with a negative iodine test   Given this information:   – The presence of Trichomonas vaginalis indicates the patient has Trichomonas vaginitis, a type of specific colpitis (inflammation of the vagina) –

Treatment for Trichomonas vaginitis typically involves antibiotic therapy directed at the Trichomonas parasite – The colposcopy findings of hazy fields on the labia warrants further investigation with a subsequent biopsy to rule out any pre-cancerous changes  

So the recommended course of action would be option A: Treatment of the specific colpitis (Trichomonas vaginitis) with metronidazole or another antiparasitic drug, along with performing a biopsy of the abnormal colposcopy findings to evaluate further.   Options B through E focus on procedures related to the cervix, but there are no abnormal cervical findings noted in the information provided, so they would not be the most appropriate first-line tactics in this case.   Hope this explanation helps! Let me know if you have any other questions.


93. A 52 y.o. patient with previously functional Class II angina complains of 5 days of intensified and prolonged retrosternal pains, decreased exercise tolerance. Angina is less responsive to Nitroglycerinum. Which of the following diagnosis is most likely?

A. IHD. Unstable angina

B. Cardialgia due to spine problem

C. IHD. Functional Class II angina

D. Myocarditis E. Myocardial dystrophy


Answer:  IHD. Unstable angina

Explanation

The key points that indicate this diagnosis:   – The patient has a history of stable angina (functional class II)   – Now the symptoms have intensified, with retrosternal chest pain for 5 days that is prolonged and less responsive to nitroglycerin (the usual angina medication)   –

Exercise tolerance has decreased, suggesting the angina has become more severe recently   These findings suggest the patient’s angina has become unstable. The features that distinguish unstable angina include:   – New onset or worsening of previously stable angina   – Angina at rest or with minimal exertion   – Angina that is resistance to or requiring increased nitroglycerin for relief   –

Decreased exercise tolerance   Options B and C (cardialgia and stable angina) do not match the current, more severe presentation. Options D and E (myocarditis and myocardial dystrophy) are less likely given the long history of stable angina with recent change to more severe symptoms, which is more suggestive of worsening ischemia and unstable angina.  

So in summary, the key clinical features – intensified chest pain, less response to medication, and decreased exercise tolerance on a background of previously stable angina – point to unstable angina as the most likely diagnosis in this patient.   Hope this explanation helps! Let me know if you have any other questions.


94. A full-term child survived antenatal and intranatal hypoxia, it was born in asphyxia (2-5 points on Apgar score). After birth the child has progressing excitability, there are also vomiting, nystagmus, spasms, strabismus, spontaneous Moro’s and Babinsky’s reflexes. What localization of intracranial hemorrhage is the most probable?

A. Subarachnoid hemorrhage

B. Small cerebral tissue hemorrhages

C. Subdural hemorrhage

D. Periventricular hemorrhages

E. Hemorrhages into the brain ventricles


Answer:  Subarachnoid hemorrhage

Explanation

– The child was exposed to antenatal and intranatal hypoxia and was born in asphyxia, suggesting a lack of adequate oxygen to the brain around the time of birth   – Periventricular intraventricular hemorrhage (PIVH) is a common complication of preterm birth and birth asphyxia, due to the fragile blood vessels in the periventricular region of the immature brain   – PIVH often occurs in the first few days after birth, which fits the child’s presenting symptoms of irritability, vomiting, nystagmus and spasms   –

Other findings like strabismus, and abnormal reflexes can also be seen with PIVH, depending on the severity and location of the hemorrhages   In contrast:   – Subarachnoid hemorrhage and subdural hemorrhage are less common in newborns, especially associated with birth asphyxia in term infants   –

Small cerebral hemorrhages and hemorrhages into the ventricles themselves would be unlikely to produce all of the described neurological signs so soon after birth   So periventricular hemorrhages, due to their location close to structures involved in motor control, ocular movements, and reflexes, best match the clinical picture presented in this newborn with a history of birth asphyxia.   Hope this explanation is helpful! Please let me know if you have any other questions.


95. The patient 25 y.o. was admitted on the 1st day of the disease with complaints of double vision in the eyes, heavy breathing. The day before the patient ate homemade mushrooms. On objective examination: paleness, widened pupils, disorder of swallowing, bradycardia, constipation are marked. What is the diagnosis?

A. Botulism

B. Yersiniosis

C. Leptospirosis

D. Salmonellosis, gastrointestinal form

E. Lambliasis


Answer: Botulism

Explanation

The key factors that point to botulism:   1. The patient had symptoms of double vision, difficulty swallowing, and difficulty breathing – all classic features of botulism poisoning.   2. The patient had eaten homemade mushrooms, which can sometimes be contaminated with Clostridium botulinum spores that produce botulinum toxin when the mushrooms are improperly preserved.  

3. On exam, the patient showed signs of paleness, dilated pupils, dysphagia (swallowing difficulty), bradycardia (slow heart rate), and constipation – all consistent with severe botulism intoxication.   4. The acute onset of symptoms within a day of eating the mushrooms also fits with botulism, which has a rapid progression once the toxin is ingested.   In contrast, the other options:   – Yersiniosis, leptospirosis, and salmonellosis would not typically cause the specific neurological symptoms seen in this case.   –


Lambliasis (giardiasis) would cause gastrointestinal symptoms but not the vision changes, difficulty swallowing and breathing, or autonomic features seen here.   So the combination of mushroom exposure, acute onset of neurological and autonomic symptoms, and specific physical exam findings point clearly to botulism as the most likely diagnosis in this patient. The features and timeline of the case match the classic presentation of botulism toxin poisoning.   Hope this explanation helps! Let me know if you have any other questions.


96. A healthy 75 y.o. woman who leads a moderately active way of life went through a preventive examination that revealed serum concentration of common cholesterol at the rate of 5,1 mmol/l and HDL (high-density lipoproteins) cholesterol at the rate of 70 mg/dl. ECG reveals no pathology. What dietary recommendation is the most adequate?

A. Any dietary changes are necessary

B. Decrease of cholesterol consumption

C. Decrease of saturated fats consumption

D. Decrease of carbohydrates consumption

E. Increase of cellulose consumption


Answer: Any dietary changes are necessary

Explanation

The key factors to consider:   – The patient is a healthy 75-year-old woman who is moderately active   – Her total cholesterol level of 5.1 mmol/L (197 mg/dL) is only slightly elevated and on the lower end of the at-risk range   – Her HDL cholesterol level of 70 mg/dL is considered normal to high, which is protective against cardiovascular disease   –

She has no signs of cardiovascular disease on ECG   Based on these factors:   – The patient’s current lifestyle seems to be supporting generally healthy cholesterol levels, with a normal HDL and only a slightly elevated total cholesterol   – In an elderly person, a total cholesterol level under 240 mg/dL is generally considered acceptable without treatment   –

Higher HDL levels help offset the possible risks of slightly elevated total cholesterol   – There are no obvious indicators that any major dietary changes are needed at this time   While recommendations to reduce saturated fat, cholesterol and carbohydrate intake can sometimes be appropriate for cholesterol management, they are not clearly warranted in this case given the patient’s:   – Generally healthy cholesterol profile – Lack of documented cardiovascular disease –

Moderately active lifestyle   So in summary, the patient’s test results do not suggest that major dietary changes are necessary at this time. Her current lifestyle and diet seem to be adequately supporting her cholesterol and cardiovascular health for a woman her age.   Hope this explanation makes sense! Let me know if you have any other questions.


97. Laparotomy was performed to a 54 y.o. woman on account of big formation in pelvis that turned out to be one-sided ovarian tumor along with considerable omental metastases. The most appropriate intraoperative tactics involves:

A. Ablation of omentum, uterus and both ovaries with tubes

B. Biopsy of omentum

C. Biopsy of an ovary

D. Ablation of an ovary and omental metastases

E. Ablation of omentum and both ovaries with tubes


Answer: Ablation of omentum, uterus and both ovaries with tubes

Explanation

The key factors:   – The patient is a 54-year-old woman undergoing laparotomy for a large pelvic mass   – The mass was found to be a unilateral (one-sided) ovarian tumor   – There are also significant omental metastases (spread of cancer to the omentum)   Given these details:   –

Ovarian cancer that has spread to the omentum is usually advanced stage disease   – When ovarian cancer is found with omental metastases, it often indicates it has spread to other intra-abdominal locations as well   – In this situation, a maximal cytoreductive surgery including removal of the omentum, both ovaries and fallopian tubes, and the uterus (total abdominal hysterectomy bilateral salpingo-oophorectomy or TAH-BSO) offers the best chance of optimal tumor resection   –

Biopsies alone (options B and C) would be insufficient given the extent of disease   – Removing only the affected ovary and omental metastases (option D) risks leaving behind microscopic tumor deposits  

So the most appropriate intraoperative tactic, to achieve the greatest tumor cytoreduction and optimize outcomes, is a total abdominal hysterectomy with bilateral salpingo-oophorectomy along with removal of the involved omentum (Option A).   Hope this explanation helps! Let me know if you have any other questions.


98. Heart auscultation of a 16 y.o. boy without clinical symptoms revealed accent of the S II and systolic murmur above the pulmonary artery. Heart sounds are resonant, rhythmic. What is the most probable diagnosis?

A. Functional murmur

B. Stenosis of pulmonary artery valve

C. Insufficiency of pulmonary artery valve

D. Nonclosure of Botallo’s duct

E. Defection of interatrial septum

Answer: Functional murmur

Explanation

The key reasons:   – The patient is a 16 year old boy with no clinical symptoms   – The only abnormal findings on exam are an accentuated pulmonary component of S2 and a systolic murmur over the pulmonic area   – The heart sounds are described as resonant and rhythmic, with no mention of abnormal sounds   These findings are classic for a functional murmur, which is:   –

A harmless heart murmur heard in anatomically normal hearts   – Most common in young, healthy individuals   – Typically has no abnormal heart sounds aside from the murmur itself   Options B through E (structural abnormalities) are less likely given:   – The patient’s age (most structural issues present earlier in childhood)   –

The lack of associated clinical symptoms   – The description of normal heart sounds other than the murmur   Therefore, in an otherwise healthy teenage boy, the most likely cause of an isolated pulmonic systolic murmur and accentuated pulmonary S2 is a functional murmur related to physiologic changes in blood flow, not an underlying structural heart defect.  

Structural issues would generally cause other physical exam abnormalities and clinical manifestations in addition to the murmur. But in this case, no such findings are present.   Hope this explanation helps! Let me know if you have any other questions.


99. A 74 y.o. patient has been suffering from hypertension for 20 years. He complains of frequent headache, dizziness, he takes enalapril. Objectively: accent of the SII above aorta, Ps- 84 bpm, rhythmic, AP- 180/120 mm Hg. What group of hypotensive medications could be additionally prescribed under consideration of the patient’s age?

A. Thiazide diuretics

B. Loop diuretics

C. β-adrenoceptor blockers

D. α-adrenoceptor blockers

E. Central sympatholytics


Answer:  Thiazide diuretics

Explanation

The key reasons:   – The patient is a 74 year old man with longstanding, uncontrolled hypertension despite being on an ACE inhibitor (enalapril)   – He is experiencing symptoms of hypertension including headache, dizziness and palpitations   –

His blood pressure is significantly elevated at 180/120 mmHg   – On exam, he has an accentuated aortic S2, likely from longstanding hypertension   – Thiazide diuretics are well-suited for the treatment of isolated systolic hypertension, which is more common in the elderly   – They are also very effective at reducing blood volume and thus lowering blood pressure, with fewer side effects compared to loop diuretics   – Thiazide diuretics have been shown to be beneficial in reducing cardiovascular events in older hypertensive patients   –

Beta blockers, alpha blockers, and central sympatholytics (options C through E) are less preferred as additional agents in this elderly patient due to higher risks of side effects like hypotension, dizziness and fatigue.  
So in summary, adding a low-dose thiazide diuretic is the best option to optimize blood pressure control in this 74 year old man with longstanding hypertension. The efficacy, safety and known benefits in older patients make thiazide diuretics the preferred additional antihypertensive for this age group.   Hope this explanation helps! Let me know if you have any other questions.


100. In treatment and prevention establishments, regardless of their organisational and proprietary form, the rights of patients should be observed. Which of these rights are the most significant?

A. The right to the protection of the patient’s interests

B. The right to the free choice

C. The right to the information

D. The right to be heard

E. The right to the protection from incompetence


Answer: The right to the protection of the patient’s interests

Explanation

Some key points:   1. All other patient rights – free choice, information, voice, and protection from incompetence – ultimately serve the overarching goal of protecting the patient’s interests and welfare.   2. Healthcare exists first and foremost to serve the patient. So the patient’s interests should be the central and guiding concern of all medical decisions and actions.  

3. Without assurance that their interests will be protected, patients may not feel comfortable exercising other rights like providing information or making choices.

The protection of interests enables all other rights.   4. Protecting patient interests means considering the patient’s whole health condition, values, and life situation, not just addressing the specific medical problem.

It means preventing and minimizing harm wherever possible.   5. Patient rights exist to empower patients and ensure the healthcare system acts in their best interests. The right to have one’s interests protected is foundational to that purpose.  

So in summary, while all the rights you listed are important, the right to have one’s welfare and interests guarded (Option A) is the most fundamental. It underpins why patient rights exist in the first place and enables the realization of all other healthcare rights.   Hope this explanation helps clarify the significance of the right to protection of interests for patients! Let me know if you have any other questions.
Join the conversation
0% Complete